The average weight of an adult one horned male rhinoceros found in Nepal is about 21 quintal.​

Answers

Answer 1

The average weight of an adult one-horned male rhinoceros found in Nepal is about 21 quintals. The scientific name for the one-horned rhinoceros is Rhinoceros unicorn is.

One-horned rhinoceroses are found in India, Nepal, Bhutan, and Indonesia. The one-horned rhinoceros is the largest animal in Nepal.

It can weigh up to 2,300 kg. Its body is covered in skin that looks like plates of armor. The one-horned rhinoceros is a herbivore, meaning it eats only plants.

They live in swamps, marshes, and forested areas near rivers, and they are excellent swimmers.

The one-horned rhinoceros has been endangered for many years due to poaching, habitat loss, and hunting.

The numbers of these animals have been increasing due to conservation efforts and strict laws in Nepal. The average weight of an adult one-horned male rhinoceros found in Nepal is about 21 quintals.

They are a protected species in Nepal, and there are several parks and conservation areas where they can be seen.

For more such questions on quintals

https://brainly.com/question/29666812

#SPJ8

Here is the complete question below:

The average weight of an adult one-horned male rhinoceros found in Nepal is about 21 quintal. Can you provide more information about the size, habitat, or any other relevant details regarding these rhinoceroses?


Related Questions

Please solve.. I will mark you brainliest

Please solve.. I will mark you brainliest

Answers

Answer:

z = -4

Step-by-step explanation:

\(6z - 16 = 2z \\ - 16 = - 6z + 2z \\ - 16 = 4z \\ - \frac{16}{4} = - 4 = z\)

what is the slope of (2,5) (-6,-3)

Answers

Answer:

m=1

Step-by-step explanation:

(2,5);(−6,−3)

(x1,y1)=(2,5)

(x2,y2)=(−6,−3)

Use the slope formula:

m=

y^2−y^1

x^2−x^1

=

−3−5/−6−2

=

−8/−8

=1

Answer:

m=1

Answer:

\(\boxed{\sf{1}}\)

Step-by-step explanation:

To solve this problem, you have to use a slope formula.

Slope formula:

\(\Longrightarrow: \sf{\dfrac{y_2-y_1}{x_2-x_1}=\dfrac{Rise}{run} }\)

\(\sf{y_2=(-3)}\\\\\sf{y_1=5}\\\\\sf{x_2=(-6)}\\\\\sf{x_1=2}\)

Rewrite the problem down.

\(\sf{\dfrac{(-3)-5}{(-6)-2} }\)

Solve.

\(\sf{\dfrac{(-3)-5}{(-6)-2}=\dfrac{-8}{-8}=\boxed{\sf{1} }\)

Therefore, the slope is 1, which is our answer.

I hope this helps you! Let me know if my answer is wrong or not.

hElP mE plS anD ty I hAd tO rEpoSt thiS :')

hElP mE plS anD ty I hAd tO rEpoSt thiS :')

Answers

Answer:

1: below its initial height.

2: if Jimmy has 20 balls and Chris take 15,how many balls does Jimmy have?

3: I would want Mrbeast to be the President because he would help everyone.

Consider the following set \( \{2,2,3,4,5,5\} \). a) How many six-digit odd numbers can be formed using these digits? b) How many even numbers greater than 500,000 can be formed using these digits?

Answers

Hence a) 60 six-digit odd numbers can be formed using these digits. b) 12 even numbers greater than 500,000 can be formed using these digits

a) Given set is {2, 2, 3, 4, 5, 5}

A number formed by these digits will be odd if and only if its unit digit is odd, i.e., 3 or 5.

The number of ways to select one of the two odd digits is 2

The other digits can be arranged in the remaining five places in 5! / (2! × 2!) = 30 ways.

So, the total number of six-digit odd numbers that can be formed is 2 × 30 = 60.

b) The number should be greater than 500,000 and should be even. The first digit has only one choice, which is 5.

The second digit has 3 choices from the set {2, 3, 4}.

The third digit has 2 choices from the set {2, 5}.

The fourth digit has 2 choices from the set {2, 5}.The fifth digit has only one choice, which is 2.

So, the total number of even numbers greater than 500,000 that can be formed using these digits is 3 × 2 × 2 × 1 = 12.

To know more about number visit:

https://brainly.com/question/24908711

#SPJ11

In performing a regression analysis involving two numerical variables, you are assuming I. the variances of Xand Yare equal. II. the variation around the line of regression is the same for each Xvalue. III. that Xand Yare independent. Select one: A. I only C B. III only C. II only D.All of these

Answers

As X and Y are two numerical variables. Because they are independent so there is no relationship between the two variables other than the linear relationship described by the regression equation. The correct answer is B. III only, that is  X and Yare independent.

When performing a regression analysis involving two numerical variables, we assume that X and Y are independent, meaning that the value of one variable does not depend on the value of the other variable. However, we do not assume that the variances of X and Y are equal or that the variation around the line of regression is the same for each X value. These assumptions are not necessary for performing a regression analysis. Violation of this assumption can lead to spurious results and incorrect inferences. So, the correct option in  B. III only.

To know more about regression:

https://brainly.com/question/14313391

#SPJ4

Sarah scored an 8/10 on the spelling test and Jonathan scored 75% of Sarah’s score on the spelling test. What is the score of Jonathan?

Answers

\(\frac{3}{5}\) is the score of Jonathan.

What is percentage?

Percentage, which may also be referred to as percent, is a fraction of a number out of 100%. Percentage means "per 100" and denotes a piece of a total amount.

To calculate the percentage , Use the percentage formula:

\(X * \frac{Percentage}{100} = Y\)

According to the question:

Sarah scored on the spelling test  = \(\frac{8}{10}\)

Jonathan scored  = 75%

By formulating both the values;

\(\frac{8}{10} * \frac{75}{100}\)

= \(\frac{3}{5}\)

Therefore , \(\frac{3}{5}\) is the score of Jonathan.

Learn more about percentage here:

https://brainly.com/question/13450942

#SPJ1

How many 3/4s are in 2 ?
1 .Write a multiplication equation and a division equation that can be used to answer the question.
2.Draw a tape diagram and answer the question. Use the grid to help you draw, if needed.

Answers

Answer:

3/4 into decimal = 0.75

0.75 X 100 = 75

2 X 100 = 200

200/75 = 2.66666666

write an explicit formula for a subscript n, the nth term of the sequence 8, 11, 14,...



please please help

Answers

The required explicit formula for the given sequence to determine the nth term is given as nth term = 8 + (n - 1)3.

What is arithmetic progression?

Arithmetic progression is the series of numbers that have common differences between adjacent values.

here,
The Sequences 8, 11, 14,...

first term a = 8
common difference = 11 - 8 = 3

Now,
The explicit formula is given as,
nth terms = a + (n - 1)d
nth term = 8 + (n - 1)3

Thus, the required explicit formula for the given sequence to determine the nth term is given as nth term = 8 + (n - 1)3.

Learn more about arithmetic progression here: https://brainly.com/question/20334860

#SPJ1

What relationship is represented by the graph

What relationship is represented by the graph

Answers

Answer:

the third one

Step-by-step explanation:

area of a circle vs. it's diameter

Whyy, do vou think, banks and other financial institutions offer cash loans to people that did not apply for it?

Answers

Answer:

because there are in need of it

Step-by-step explanation:

yes

find the volume of the solid that is generated when the given region is revolved as described.the region bounded by f(x) = e^-x and the x-axis on [0,ln 14] is revolved about the line x=ln 14.The volume is (Type an exact answer.

Answers

Thus,  the volume of solid generated when the region bounded by f(x) = e^(-x)  is approximately 24.7842 cubic units.

To find the volume of the solid generated when the region bounded by f(x) = e^(-x) and the x-axis on [0, ln 14] is revolved about the line x = ln 14, we will use the disk method. The formula for the disk method is:

V = π * ∫[a, b] (R(x))^2 dx

where V is the volume, a and b are the bounds, R(x) is the radius function, and dx is the infinitesimal change in x. In this case, a = 0 and b = ln 14, and R(x) = e^(-x).

The radius function R(x) can be found by subtracting the revolving axis value (ln 14) from the x-value:
R(x) = ln 14 - x

Now we can set up our integral:
V = π * ∫[0, ln 14] (ln 14 - x)^2 * e^(-x) dx

To find the volume, we will need to evaluate this integral. This requires integration by parts, and can be quite complex to calculate manually. It's recommended to use an advanced calculator or software like WolframAlpha to evaluate the integral. The result is:

V ≈ 24.7842

So, the volume of the solid is approximately 24.7842 cubic units.

know more about the integration by parts

https://brainly.com/question/30215870

#SPJ11

If
C
=
s

5
C=s−5 and
D
=
s

8
,
D=s−8, find an expression that equals
C
+
2
D
C+2D in standard form.

Answers

9514 1404 393

Answer:

  C +2D = 3s -21

Step-by-step explanation:

Use the substitution property of equality.

  C = s -5

  D = s -8

Then the value of your expression is ...

  C +2D = (s -5) +2(s -8)

  = s -5 +2s -16

  C +2D = 3s -21

What is the answer to this equation: -13<2x+5<15

Answers

I believe the answer is

-9

find the range of the line

find the range of the line

Answers

Answer:

(-10,3),(9,0)

Step-by-step explanation:

The graph of f(x)=x^2
has been translated 4 units down. whats the equation

Answers

Answer:

the new equation is f(x)=x^2-4  

Step-by-step explanation:

4(2x – 5) = 4

Part A: How many solutions does this equation have? (4 points)

Part B: What are the solutions to this equation? Show your work.

Answers

The equation has one solution

The solution to the equation is  x = 3

How to solve the equation

4(2x – 5) = 4

To solve this, we would first have to open the bracket.

4 * 2x - 4*5 = 4

8x - 20 = 4

We would want to have the variable x stand on its own

so we take like terms

8x = 4 + 20

8x = 24

To get the value of x divide through the equation by 8

x = 24 / 8

x = 3

The solution is just one at x = 3

Read more on equations here: https://brainly.com/question/2972832

#SPJ1

QUICK PLEASE!!!
Consider f(x) Equals StartFraction 8 (x minus 1) Over x squared + 2 x minus 3 EndFraction.

Which statements describe the existence of vertical asymptotes at x = –3 and x = 1?

Answers

Answer:

A

Step-by-step explanation:

edge 2021

Answer:

It is A

Step-by-step explanation:

I got it right on the test Edge 2022.

11 Finding a difference quotient for a linear or quadratic function V Find the difference quotient f(x)=-3x²-2x+5 Simplify your answer as much as possible. f(x +h)-f(x) h f(x+h)-f(x) h = ( where h#0,

Answers

The difference quotient for the given function is 9 -2/h.

The difference quotient for the given function can be calculated as:

[f(x+h) - f(x)]/h

= [(3(x+h)² - 2(x+h) + 5) - (3x² - 2x + 5)]/h

= (3x² + 6xh + 3h² - 2x - 2h + 5 - 3x² + 2x - 5)/h

= (6xh + 3h² - 2h)/h

= (6x + 3h -2)/h

Simplifying the expression further, we get:

(6x + 3h -2)/h = 6 + 3h/h -2/h

= 6 + 3 -2/h

= 9-2/h

Therefore, the difference quotient for the given function is 9 -2/h.

Learn more about the quadratic function here:

https://brainly.com/question/29775037.

#SPJ4

"Your question is incomplete, probably the complete question/missing part is:"

Find the difference quotient [f(x+h)-f(x)]/h, where h≠0, for the function below.

f(x)=3x² -2x+5. Simplify your answer as much as possible.

Apply the distributive property to create an equivalent expression.
6(5x -3) =6(5x−3)=

Answers

The correct answer is =0

Answer:

30x-18

Step-by-step explanation:

Multiply 6 to both 5x and -3. The equivalent expression you will get is 30x-18.

The question i stated in the screenshot.
I just need to find the answer for the green box [?]
It isn't 1-10 because I have already gotten that wrong.


Hurry Please!

The question i stated in the screenshot.I just need to find the answer for the green box [?]It isn't

Answers

Answer:

The number in the green box should be, 11

in scientific notation, we get the number,

\((9.32)(10)^{11}\)

Step-by-step explanation:

Answer:

11

Step-by-step explanation:

Look at the blue number 9.32. The decimal point is in between the 9 and the three. On the problem the decimal point is at the very end after the last zero, all the way to the right. It is understood, that means it's not written. So how many hops does it take to get the decimal from the end all the way over to in between the nine and the three? It takes 11 moves. The exponent is 11

Please help me !! Show work if possible

Please help me !! Show work if possible

Answers

Answer:

x = -(27/8)

Step-by-step explanation:

8x + 5y -z = 0

y=4 and z = -7

8x + 5(4) -(-7) = 0

8x + 20 +7 = 0

8x = -27

x = -(27/8)

x = -24 and 3/8

An item on sale costs %25 of the original price. The original price was $95

Answers

Sale Price = $71.25 (answer). This means the cost of the item to you is $71.25. You will pay $71.25 for a item with original price of $95 when discounted 25%. In this example, if you buy an item at $95 with 25% discount, you will pay 95 - 23.75 = 71.25 dollars.
The answer should be 71 dollars

the frequency of countries with both a population density of fewer than 100100100 per \text{km}^2km 2 start text, k, m, end text, squared and a medium population among all countries in north america is 0.1740.1740, point, 174. how many countries have both a population density of fewer than 100100100 per \text{km}^2km 2 start text, k, m, end text, squared and a medium population? choose 1 answer: (choice a) a

Answers

The number of countries that have both a population density of fewer than 100,100,100 per square kilometer and a medium population among all countries in North America is approximately 174.

To understand the answer, we can interpret the given frequency as a probability. The frequency of 0.174 indicates that out of all countries in North America, 17.4% (or 0.174 as a decimal) have both a population density of fewer than 100,100,100 per square kilometer and a medium population.

To calculate the number of countries, we would multiply this probability by the total number of countries in North America. However, the total number of countries in North America is not provided in the given information. Therefore, without knowing the total number of countries, we cannot provide an exact count of the countries that meet the specified criteria.

In summary, the given information states that the frequency of countries with both a population density of fewer than 100,100,100 per square kilometer and a medium population among all countries in North America is 0.174. However, without the total number of countries in North America, we cannot determine the exact count of countries that meet the specified criteria.

Learn more about probability here:

https://brainly.com/question/31828911

#SPJ11

what is the length of the base of a triangle with height 22.2 mm and area 293.04 mm squared

Answers

Answer:

b = 26.4 mm

Step-by-step explanation:

The formula for area of a triangle is A = 1/2bh

By plugging in what we know, we have 293.04 = 1/2b * 22.2

13.2 = 1/2b

26.4 = b

5x - y = -7
X + y = -5

Answers

Answer:

x = -2

y = -3

Step-by-step explanation:

5x - y = -7

x + y = -5

6x = -12

x = -2

x + y = -5

-2 + y = -5

y = -5 + 2

y = -3

Answer:

(-2,-3)

Step-by-step explanation:

x + y = -5

y = -x - 5

5x - (-x - 5) = -7

5x + x + 5 = -7

6x = -12

x = -2

x + y = -5

-2 + y = -5

y = -3

Maggie is working at a store that pays by the hour and by commission (pay for how much you sell). Maggie wants to go this weekend to the lake with her friends but she needs to make at least $225 today. She gets paid $15 per hour plus $25 for every sale she makes. What are all the possible values of the number of sales that Maggie can make to go to the lake if she is scheduled to work from 8am until 4pm?​

Answers

Maggie can make anywhere from 5 to 4 sales to earn at least $225 and go to the lake with her friends.

Maggie gets paid $15 per hour plus $25 for every sale she makes. The number of sales she makes can be represented by x.

In order to calculate Maggie's earnings in terms of commission, we can use the equation 25x.

To calculate Maggie's earnings in terms of hourly pay, we can use the equation 15(8), since she works from 8am until 4pm, which is 8 hours. This simplifies to 120.The total amount Maggie earns can be represented by the equation:

Total earnings = 25x + 120

To find the minimum number of sales Maggie needs to make to earn at least $225, lets set up the inequality:

25x + 120 ≥ 225

Subtracting 120 from both sides, we get:

25x ≥ 105

Dividing both sides by 25, we get:

x ≥ 4.2

Maggie cannot make a fraction of a sale, so we can round up to find the minimum number of sales she needs to make, which is 5 sales.

To find the maximum number of sales Maggie can make, lets consider the fact that she is scheduled to work from 8am until 4pm, which is 8 hours. If she makes 0 sales, she will earn $120 (her hourly pay for 8 hours of work).

To find the maximum number of sales, we can set up the equation:25x + 120 ≤ 225

Subtracting 120 from both sides, we get:

25x ≤ 105

Dividing both sides by 25, we get:

x ≤ 4.2

Maggie cannot make a negative number of sales, so we can round down to find the maximum number of sales she can make, which is 4 sales.

To know more about inequalities please visit :

https://brainly.com/question/30238989

#SPJ11

The possible values of the number of sales that Maggie can make to go to the lake are 5 and 4.

Given:

Maggie gets paid $15 per hour plus $25 for every sale she makes.

She needs to make at least $225 today.

She is scheduled to work from 8 am until 4 pm.

To find:

All the possible values of the number of sales that Maggie can make to go to the lake.

Solution:

Let's consider x to be the number of sales that Maggie makes.

To determine the minimum amount she needs to earn:

Her hourly wage for 8 hours of work = $15 × 8 = $120

Total earnings that she needs = $225 - $120 = $105

If y is the number of sales she needs to make to earn $105, then:

$25y = $105

Dividing both sides by $25, we get:

y = 4.2

This means she needs to make at least 5 sales.

Let's calculate the maximum number of sales that she can make. If she has to earn $240 for 8 hours of work:

Total earnings required = $240 - $120 = $120

$25y = $120

Dividing both sides by $25, we get:

y = 4.8

This means the maximum number of sales she can make is 4.

As such, the possible values of the number of sales that Maggie can make to go to the lake are 5 and 4.

To know more about  the number of sales, Visit :

https://brainly.com/question/31509025

#SPJ11

Please help! Correct answer only, please!

Fill in the missing numbers to complete the pattern:


___, ____, 5.1 , 5.2 , 5.3 , 5.4

Please help! Correct answer only, please!Fill in the missing numbers to complete the pattern:___, ____,

Answers

Answer:

4.9,5.0

Step-by-step explanation:

Thier is two open spaces and the decimals are decreasing by 1 tenth. So if 1 tenth is being decreased the order would be 4.9, 5.0, 5.1, 5.2, 5.3, 5.4.

Answer:

4.9, 5 , 5.1 , 5.2 , 5.3 , 5.4

Step-by-step explanation:

5.4-5.3 = 0.1

5.3-5.2 = 0.1

5.2-5.1 = 0.1

then the pattern is : adding 0.1 to the number to get the next number

5.1 - 0.1 = 5

5 - 0.1 = 4.9

*WILL GIVE BRAINLIEST FOR BEST ANSWER*


Find the missing angle in the triangle.





Question 1 options:

A. 90°


B. 25°


C. 108°


D. 130°

*WILL GIVE BRAINLIEST FOR BEST ANSWER* Find the missing angle in the triangle.Question 1 options:A. 90B.

Answers

Answer:

90 degrees

Step-by-step explanation:

A triangle's degrees add up to 180. You are already given 2 of the. Let x be the 3rd angle. Here is the equation: 29+61+x=18. x=90.

Hope this helped!

Answer:

A.90°

Step-by-step explanation:

A triangle's interior angles are equal to 180°.

So since two angles are given all you need to do is add the given angles and subtract the sum from 180°.

61°+29°=90°

180°-90°=90°

The missing angle m<R is equal to 90°

Your answer is: A.90°

Hope this helps! :)

Mr Yadav buys a radio for Rs 640 and sells it for Rs 672 Find it's profit​

Answers

Answer:

hope it is helpful to you

Mr Yadav buys a radio for Rs 640 and sells it for Rs 672 Find it's profit

Can someone help me with this ??????????

Can someone help me with this ??????????

Answers

Answer:

73%

Step-by-step explanation:

Other Questions
Problem 1 Write code to print the following shape to the console: Tip: you can do it in three lines of output instead of a single function Tip: Just find a way to print result, you do not need loop for this problem. But It is okay if you use a loop. You are testing the null hypothesis that there is no linearrelationship between two variables, X and Y. From your sample ofn=18, you determine that b1=4.6 and Sb1=1.5. Construct a95% confidence int Incompatibilities of Parenteral Formulations:Physical Incompatibility sources A square has a area of 4m^2. What is the length of each side? Why was the South upset with the North for sending supplies to Fort Sumter?A. It was a Union fort in a border state.B. It was a Union fort in southern territory.C. It was a Confederate fort in a border state.D. It was a Confederate fort in southern territory. Which of the following compounds would be expected to form intermolecular hydrogen bonds in the liquid state? (a) CH3OCH3 (dimethyl ether), (b) CH4, (c) HF, (d) CH3CO2H (acetic acid), (e) Br2, (f) CH3OH (methanol) Why is it misleading to compare a companys financial ratios with those of other firms that operate within the same industry? How are the perspectives of the main character and another character different?story is thank you ma'm nurses must maintain the privacy of clients. which example is a breach in privacy and would pose an ethical problem? the young of endangered/threatened species can be protected from predation pressures. (click to select) Cindy earns $10 per hour, and Claire earns $12 per hour. If they each work h hours, how much do they earn together? Determine if the following ratios form a proportion.[tex]\frac{4}{16}[/tex] [tex]\frac{5}{20}[/tex]- Yes, these ratios are proportional.- No, these ratios are not proportional. If u play volleyball answer this plz Make sure T is in Kelvin! If it is in C then use K = C +273Example 1: If a 2.0 L sample of air is collected at 298K, but its temperature falls to -25C, what is the finalvolume of the sample? A political consequence of sustainability describes a change in the way people or businesses are governed. Which statement best describes a political consequence of sustainable waste management?A. Businesses are penalized for improper waste clean-up.B. Incinerators release fewer emissions into the atmosphere.C. Transporting waste to other countries saves money for landfills.D. Ecosystems are saved when less land is used for landfills. The dot plot shows the number of hours, to the nearest hour, that a sample of 5th- and 7th-grade students spend watching television each week. How do the medians compare?The 5th-grade range is .The 7th-grade range is .The ratio of variation of 5th grade to 7th grade is Compare and contrast the political insurance and commitmenttheories that explain why new democracies gravitate towards theestablishment of the power of judicial review in theirconstitutions. 1. To solve the equation with algebra tiles, first remove [answer]. Then divide each side into [answer].2. The solution is x= [answer]. sally invests 8000 in a savings accountthe account pays 2.8% compound interest per yearwork out the value of her investment after 4 yearsgive your answer to the nearest penny 29 yo F presents with amenorrhea for the past 6 months. She has a history of occasional palpitations and dizziness. She lost her fiance in a car accident What is the most likely diagnosis?